LSAT and Law School Admissions Forum

Get expert LSAT preparation and law school admissions advice from PowerScore Test Preparation.

User avatar
 Dave Killoran
PowerScore Staff
  • PowerScore Staff
  • Posts: 5848
  • Joined: Mar 25, 2011
|
#59497
Complete Question Explanation
(The complete setup for this game can be found here: lsat/viewtopic.php?t=26462)

The correct answer choice is (A)


As with any List question, simply apply the rules to the answer choices. Given the presentation of the answer choices in this problem (the test makers have made them somewhat difficult to read), the easiest order of attack is to apply the second rule, then the first rule, then the third rule, and finally the last rule.

Answer choice (A): This is the correct answer choice.

Answer choice (B): This answer choice is incorrect because the first rule specifies that T cannot play in the third game.

Answer choice (C): This answer choice is incorrect because it violates both the third and fourth rules. The third rule is violated because N appears in two games; the fourth rule is violated because there is no SOS block.

Answer choice (D): This answer choice is incorrect because N can only play in one game, yet the answer features N in two games.

Answer choice (E): This answer choice is incorrect because L must play in the fourth game.

Get the most out of your LSAT Prep Plus subscription.

Analyze and track your performance with our Testing and Analytics Package.